11+(-3)-1/8j-3/8j+7=

Will give reward if right

Answers

Answer 1

Answer:

J = 7.5

Step-by-step explanation:

* This is going off of the estimate that the = sign means the equation is = to 0, as there is no value after the equal sign. *

First, start off by adding all of the like numbers.

11 + (-3) + 7 = 8 + 7 = 15

- 1/8 j - 3/8 j  = - 4/8 j.

Combining those two sums, we get:

15 - 4/8 j = 0

Subtract 15 from both sides,

-4/8 j = -15

Simplify -4/8 to -1/2, and then to -0.5

-0.5j = -15

Divide.

-15/-0.5 = 7.5

When you divide a negative number by another negative number, you get a positive. Therefore, the answer is J = 7.5

If the number after the = sign IS NOT 0, then when you get to 15 - 0.5j, just do the steps but with a number that isn't 0.


Related Questions

given f (x) = 5x + 1, find f (-3)

Answers

Answer:

f(- 3) = - 14

Step-by-step explanation:

To evaluate f(- 3), substitute x = - 3 into f(x), that is

f(- 3) = 5(- 3) + 1 = - 15 + 1 = - 14

These fraction are part of a list of students quiz scores. Choose ALL scores that are equal to 80%

A.) 8/20
B.) 8/10
C.) 24/30
D.) 16/20
E.) 16/30

Answers

Answer:

B and D

Step-by-step explanation:

I'm pretty sure but definitely B

DO IT ASAP ONLY 3 PROBLEMS Please find x thanks!!! (There’s a link)

Answers

1.) x = 3.5

2.) x = -40

3.) x = -10

. Austin wants to buy some pens and pencils. He has $30 in his bag. Each pen costs $5 and each pencil costs $1.50. He needs 5 more pencils than pens.Part A how many pencils can $ustin buy at most" Write your answer and your work or explanation in the space below. Part If Austin had $40 in his bag, how many pens could Austin buy at most" Explain how you found the answer in the space below.

Answers

Answer:

3 pens

Step-by-step explanation:

Total expenditure (Price x Quantity) on pens & pencils = $30

Let pen quantity be = x , Let pencil quantity be = y

Given : Pen (x) price = 5 , Pencil (y) price = 1.5 , Pencils (y) = Pen (x) + 5

So, 5x + 1.5y = 30

5x + 1.5 (x + 5) = 30  → 5x + 1.5x + 7.5 = 30  → 6.5x + 7.5 = 30

6.5x = 30 - 7.5  → 6.5x = 22.5

x = 22.5 / 6.5 → x = 3.46  ≈ 3 pens

What is the third term of the expression
4x+8y+12xy

Answers

Answer:

12xy

Step-by-step explanation:

Each term is separated by a function sign. In this case, an addition symbol.

Someone please help I absolutely suck at these

Answers

Answer:

I believe it is C

Step-by-step explanation:

y/2 = 9 What is the value of y that satisfies the equation

Answers

Answer:

y=18

Step-by-step explanation:

pls give brainliest

Antwaun waters his lawn 3 times a week. If he watered his lawn 24 times in all, which equation could be used to find how many weeks he has been watering his lawn?

(It's not 8, I know that) PLEASE HURRY

Answers

Answer:

Antwanun waters his lawn 3 times a week, he watered his lawn 24 . So what you will need to do is divide 3 in 24 to find how many weeks he has been watering his lawn, You get 8. Mr. Antwanun has been watering for 8 weeks. Equation- Antuwantun waters his lawn 3 times a week so if he watered his lawn 24 times, 3 times a week he would have been watering his lawn for 8 weeks.

Answer:

3x=24

Step-by-step explanation:

3 times a certain amount equals 24 times in total

9x + 6 – 4x – 1x + 1 – 10

Answers

Answer:

4x-3

Step-by-step explanation:

9x+6-4x-1x+1-10

9x-4x-1x+7-10

5x-1x+7-10

4x-3

4x-3 is your answer hope it helps

A cyclist estimates that he will bike 80 miles this week. He actually bikes 75.5 miles. What is the percent error of the cyclist's estimate? Round the percent to the nearest hundredth if necessary.

Answers

Answer:

6%

Step-by-step explanation:

Estimated distance = 80 miles

Actual distance covered = 75.5 miles

We need to find the percent error of the cyclist's estimate. Its formula is given by :

[tex]\%=\dfrac{|\text{actual value-estimated value}|}{\text{actual value}}\times 100[/tex]

Put all the values,

[tex]\%=\dfrac{80-75.5}{75.5}\times 100\\\\=5.96\%[/tex]

or

= 6%

So, the percentage error is 6%.

Answer:

5.96

Step-by-step explanation:

1. (5pt) In a recent New York City marathon, 25,221 men finished and 253 dropped out. Also,

12,883 women finished and 163 dropped out. We want to use a 0.01 significance level to test

the claim that the rate of those who finish is the same for men and women.

a) Test the claim using a hypothesis test (identify the null hypothesis, alternative hypothesis,

test statistic, critical value (or P-value) and conclusion)

b) Test the claim by constructing an appropriate confidence interval.

Answers

Answer:

(a) Explained below.

(b) The 99% confidence interval for the difference between proportions is (-0.00094, 0.00494).

Step-by-step explanation:

The information provided is:

n (Men who finished the marathon) = 25,221

n (Women who finished the marathon) = 12,883

Compute the proportion of men and women who finished the marathon as follows:

[tex]\hat p_{1}=\frac{25221}{25221 +253}=0.99\\\\\hat p_{2}=\frac{12883 }{12883+163}=0.988[/tex]

The combined proportion is:

[tex]\hat P=\frac{X_{1}+X_{2}}{n_{1}+n_{2}}\\\\=\frac{25221+12883}{38520}\\\\=0.989[/tex]

(a)

The hypothesis is:

H₀: The rate of those who finish the marathon is the same for men and women, i.e. p₁ - p₂ = 0.

Hₐ: The rate of those who finish the marathon is not same for men and women, i.e. p₁ - p₂ ≠ 0.

Compute the test statistic as follows:

[tex]Z=\frac{\hat p_{1}-\hat p_{2}}{\sqrt{\hat P(1-\hat P)\cdot [\frac{1}{n_{1}}+\frac{1}{n_{2}}]}}[/tex]

   [tex]=\frac{0.99-0.988}{\sqrt{0.989(1-0.989)\times[\frac{1}{25474}+\frac{1}{13046}]}}\\\\=1.78[/tex]

Compute the p-value as follows:

[tex]p-value=2\times P(Z>1.78)\\\\=2\times 0.03754\\\\=0.07508\\\\\approx 0.075[/tex]

The p-value of the test is more than the significance level. The null hypothesis was failed to be rejected.

Thus, concluding that the rate of those who finish is the same for men and women.

(b)

Compute the 99% confidence interval for the difference between proportions as follows:

The critical value of z for 99% confidence level is 2.58.

[tex]CI=(\hat p_{1}-\hat p_{2})\pm z_{\alpha/2}\cdot\sqrt{\frac{\hat p_{1}(1-\hat p_{1})}{n_{1}}+\frac{\hat p_{2}(1-\hat p_{2})}{n_{2}}}[/tex]

     [tex]=(0.99-0.988)\pm 2.58\times\sqrt{\frac{0.99(1-0.99)}{25474}+\frac{0.988(1-0.988)}{13046}}\\\\=0.002\pm 0.00294\\\\=(-0.00094, 0.00494)\\\\[/tex]

Thu, the 99% confidence interval for the difference between proportions is (-0.00094, 0.00494).

What is a good estimate for 1,710 2,740 4.322 5,700 7,810 6,395

Answers

Answer: If you want to round is so you have a small amount of decimals, you could round to the nearest tenths which would give you 17,102,740 4.3

Step-by-step explanation:

Which values for x make this inequality true?

−15x+4≤39

Answers

Answer:

x ≥ -7/3

Step-by-step explanation:

General Formulas and Concepts:

Pre-Algebra

Order of Operations: BPEMDAS

Brackets

Parenthesis

Exponents

Multiplication

Division

Addition

Subtraction

Left to Right

Equality Properties

Step-by-step explanation:

Step 1: Define Inequality

-15x + 4 ≤ 39

Step 2: Solve for x

Subtract 4 on both sides:                     -15x ≤ 35

Divide -15 on both sides:                      x ≥ -7/3

Here we see that any value x greater than or equal to -7/3 would work as a solution to the inequality.

Identify the equation in slope-intercept form for the line containing the point (2,4) and perpendicular to y=1/2x+1/2.

Answers

Answer:

 y = − 2 x

Step-by-step Explanation:

Y + 2 = − 2 ⋅ ( − 1 )  Write in   y = m x + b  form.

The equation of a perpendicular line must have a slope that is the negative reciprocal of the original slope.

m  in perpendicular  = − 1 2

(1,-2),y=(1)/(2)x+4

I hope I helped!

Answer:

y = -2x + 8

Step-by-step explanation:

For this problem we need to consider that the equation of a line perpendicular to another line will have a "negative reciprocal" slope so that when they cross, it forms a 90-degree angle, hence perpendicular.  Additionally, we will need to use the point-slope form and re-write it into the slope-intercept form to incorporate (2,4) into our equation.  With these things in mind, let's do that.

y = (1/2)x + (1/2)

We want a "negative reciprocal" so:

y = -2x + (1/2)

Since -2 is the "negative reciprocal" of 1/2.

And now we want to use the point-slope form to incorporate (2,4)

y - y0 = -2 ( x - x0 )

Where, y0 = 4 and x0 = 2.

y - y0 = -2 ( x - x0 )

y - 4 = -2 ( x - 2 )

y - 4 = -2x + 4

y = -2x + 8

So, the equation that is perpendicular to y = (1/2)x + (1/2) and includes the point (2,4) is y = -2x + 8.

Cheers.

I beg you to help me .

Answers

Answer:

16+3=y

y=19

Step-by-step explanation:

A water tank holds 693 gallons but is leaking at a rate of 4 gallons per week. A second water tank holds 924 gallons but is leaking at a rate of 7 gallons per week. After how many weeks will the amount of water in the two tanks be the same?

Answers

Answer:

Simple....

you have: one is leaking 3 gallons per week and holds 256 gallons; whereas, the other one is leaking 5 gallons per week and holds 384 gallons.

Set up both the equations....

In y=mx+b form--->>>

y=-3x+256

y=-5x+384

To figure out when they will be equal..set the equations equal to each other....(remember to isolate the variable)

-3x+256=-5x+384

-3x+256=-5x+384

     -256        -256

-3x=-5x+128

-3x=-5x+128

+5x+5x

2x=128

x=64

After 64 weeks the amount of water in both tanks will be the same.

Thus, your answer.

Solve by using substitution. Express your answer as an ordered pair.

Answers

(7,4) hope that helps!

If f(x) = 4-x* and g(x) = 6x, which expression is equivalent to (g -)(3)?
O 6-3- (4 + 3)²
O6-3- (4- 3)
O 6(3) -4 +32
O 6(3) –4 -32

Answers

Answer:

The expression that equivalent to (g - f)(3) is 6(3) - 4 + 3² C

Step-by-step explanation:

Let us solve the question

f(x) = 4 - x²

g(x) = 6x

→ Find at first (g - f)(x) by subtracting f(x) from g(x)

∵ (g - f)(x) = 6x - (4 - x²)

→ Remember (-)(+) = (-) and (-)(-) = (+)

∴ (g - f)(x) = 6x - 4 + x²

→ Substitute x by 3 to find (g - f)(3)

∵ (g - f)(3) = 6(3) - 4 + (3)²

(g - f)(3) = 6(3) - 4 + 3²

The expression that equivalent to (g - f)(3) is 6(3) - 4 + 3²

The Historic Triangle in Virginia connects Jamestown, Williamsburg, and Yorktown. The interior angle at Williamsburg is 120°. The interior angle at Jamestown is twice the measure of the interior angle at Yorktown. Find the measures of the interior angles at Jamestown and Yorktown.

Answers

Answer: Interior angles at Jamestown and Yorktown are 40° and 20°, respectively

Step-by-step explanation: Jamestown, Williamsburg and Yorktown form a triangle. The sum of the interior angles of a triangle is 180°.

Suppose interior angle at Yorktown is x. As Jamestown is twice the measure at Yorktown, it means this interior angle is 2x.

Then, the measures will be:

2x + x + 120 = 180

3x = 60

x = 20

Angle at Yorktown is 20°.

So, angle at Jamestown is

2x = 2.20 = 40°

The measures of the interior angles at Jamestown and Yorktown are 40° and 20°, respectively.

Select all the true statements.
- 1.9 is to the left of -1.48.
-1.9 is to the right of -1.48.
1.9 is closer to 0.
-1.48 is closer to 0.
-1.48 is less than – 1.9.
-1.48 is greater than -1.9.

Answers

Answer:

-1.9 is to the left of -1.48.

-1.48 is closer to 0.

-1.48 is greater than -1.9.

Answer:

statements 1,4,5

Step-by-step explanation:

help plzz Charli pays $18.00 for 5 t-shirts he needs your help to answer the following questions.​

Answers

Answer:

What do you but in the chart? Also, part 2 is 72

Step-by-step explanation:

Answer:

72

Step-by-step explanation:

If each t-shirt costs 3.6 dollars then you would have to multiply 3.6 by 20 shirts which will equal to 72. So Charlie has to pay 72 dollars for 20 shirts.

Penelope earned $50 in May. She earned $60 in June.
What is the percent increase or decrease in Penelope's monthly earnings?

A. 30% decrease
B. 25% increase
C. 20% decrease
D. 20% increase​

Answers

b)25%

espero ter ajudado

ANSWER ASAP FOR BRAINLEST!!!!!!!!!!!!!!!!!!!
In how many years will $12,000 yield an interest of $4,800 at 10% simple interest? *
A. 25 years
B. 0.04 year
C. 4 years
D. 0.25 years or 3 months

Answers

Answer:

Time= 4 years

Step-by-step explanation:

Giving the following information:

Principal (P)= $12,000

Interest rate (r)= 10% = 0.10

Interest earned (I)= $4,800

To calculate the time required to gain $4,800 interest, we need to use the following formula:

T= I / (P*r)

T= 4,800/(12,000*0.1)

T= 4 years

What is the value for 3-2m<7

Answers

Answer:

m>−2

Step-by-step explanation:

3-2m<7

subtract 3 from both sides

-2m<4

divide by 2

m>−2

*PLS HELP ME ASAP 100 POINTS* There are 49 servings in a box of cereal. Each serving size is 1/7 of a cup of cereal. If one person eats 1 cup and another person eats 4 cups, how many cups are left in the box of cereal?

Answers

Answer:

2 cups left

Step-by-step explanation:

Write two hundred thousand and fifty seven in figures

Answers

200,057
This is the how write

is this proportional relationship? please help!!

Answers

Answer:

yes, the slope is the same throughout the table

Step-by-step explanation:

Answer:

Yes, the graph shown is a proportional relationship.

Step-by-step explanation:

If you divide the distance (6) by the time (1) you get the same thing (6). Repeat for all the terms in the graphs. Since 6/1 is 6, 12/2 is 6, 18/3 is 6 and 24/4 is 6, The relationship is proportional.

The greater of two integers is 10 more than twice the smaller integer. The sum of the two integers is -8. Find the integers.

Answers

Answer:

-6, -2

Step-by-step explanation:

Find the slope of the line passing through the points (-7, -4) and (8,8).

Answers

Answer:

Hope its work for you

Step-by-step explanation:

As we know that,

slope=m=(Y2-Y1)/(X2-X1)

As given values

take,   (X1,Y1)=(-7,-4)

           (X2-Y2)=(8,8)

hence   X1=-7  ,  Y1= -4   ,  X2=8  , Y2= 8

putt the values in the above equation of slope we get ,

m=(8-(-4))/(8-(-7)

m=12/15    Ans

Answer:

0.8

Step-by-step explanation:

Slope=(y2-y1) (8--4) (12)

(x2-x1)= (8--7)= (15)= 0.8

What is the general form of the equation of the line shown?

Answers

Answer:

the general form of the equation of the line will be:

[tex]y-x=2[/tex]

Step-by-step explanation:

Finding the slope:

Taking two points from the line as shown in figure

(-2, 0)(0, 2)

Finding the slope between (-2, 0) and (0, 2)

[tex]\mathrm{Slope}=\frac{y_2-y_1}{x_2-x_1}[/tex]

[tex]\left(x_1,\:y_1\right)=\left(-2,\:0\right),\:\left(x_2,\:y_2\right)=\left(0,\:2\right)[/tex]

[tex]m=\frac{2-0}{0-\left(-2\right)}[/tex]

[tex]m=1[/tex]

Finding the y-intercept

We know that the y-intercept can be calculated by setting x=0

From the figure, it is clear that at x=0, y=2

Thus, the y-intercept is (0, 2)

We know that the slope-intercept form of the equation line is:

[tex]y=mx+b[/tex]

where m is the slope and b is the y-intercept

As we have already determined the slope = m = 1 and the y-intercept b=2.

Substituting the values in the slope-intercept form of the equation line

[tex]y=mx+b[/tex]

[tex]y=(1)x+2[/tex]

[tex]y=x+2[/tex]

Writing the equation in the standard form form

As we know that the equation in the standard form is

[tex]Ax+By=C[/tex]

where x and y are variables and A, B and C are constants

As we already know the equation in slope-intercept form

[tex]y=x+2[/tex]

so just simplify the equation to write in standard form

[tex]y-x=2[/tex]

Thus, the general form of the equation of the line will be:

[tex]y-x=2[/tex]

Other Questions
The least common denominator of the fractions 1/3, 1/6 and 1/2 is 6. How are 12 and 24 related to this least common denominator? Use the functionsf(x) or g(x) toanswer thefollowing:Function f(x) has the equation f(x) = 3x 4.Function g(x) has the table of values shownbelow.Which y-interceptis smaller and howdo you know?x|g(x)0] 43 56 69 714 How is a theme different from a main idea? Find the 75th term of the following arithmetic sequence 16,25,34,43 determine how many solutions if correct bainliest There are 10 fruits in a basket and 5 of them are apples. Write the ratio in simplest form. pls help me with this thank u You do 2/3 of your math homework problems on Saturday. What fraction of your math problems must you do on Sunday in order to finish your homework? What is the slope of the line passing through the points (-1, 19) (-2, -7)??? M=? solve 28m+21=413 thanks! Help! Write the equation of the line fully simplified slope-intercept form. Write an inequality torepresent a number is atmost two Please help me with this question David wants to buy a new tablet computer. He looks at a sales ad and sees that a computer store is selling a tablet computer for $499 which is 1/3 off the regular price.Part A:Write an equation that represents the situation. Let x = the original cost of the tablet.Part B:Find the original cost of the tablet computer. find the slope of the two points:(9,-6) and (7,-3) Observe an ecosystem near your home and describe the biodiversity of your chosen ecosystem. PLZZZ HELPP idc if its from a photo just Helpp mee Three American presidents, Andrew Johnson, Bill Clinton, and Donald Trump, have been impeached. Impeachment is the investigation that takes place when an elected official is accused of illegal activity. Sb +Cl2 SbCl3Balance chemical equation A manufacturing company that produces a single product has provided the following data concerning its most recent month of operation. The total contribution margin for the month under variable costing is Explain in detail the events that led to the decree condemning Louis XVI to death.